Vous êtes sur la page 1sur 66
Ex. SoLUcionNARIO www. elsoluctonario. Het BE ststorss EER wrcenrscie | rotor ron Encuentra en nuestra pagina los Textos Universitarios que necesitas! Libros y Solucionarios en formato digital El complemento ideal para estar preparados para los exdmenes! Les Solucionarios contienen TODOS los problemas del libro resueltos yexplicados paso a paso de forma clara.. Visitanos para descargarlos GRATIS! Descargas directas mucho més facile: WWW ELSOLUCIONARIO NET Biology Investigacion Operativa mputer Science Matematicas Avanzadas Physics Estadistica Chemistry Geometria “ Math 8 wie Céleulo ronica ircult usiness - Economia Analisis Numérie Civil Engineering Equaciones Diferenciales Electrical Engineering Algebra Electromagnetismo a) Determine the function f(#) whose Fourier transform is shown in figure P-3.1.a. b) Determine the function f(t) whose Fourier transform is shown in figure P-3.1.b, ©) Sketch f(t) and g(t) near t = 0. What effects does the phase has on the symmetry of this waveform? rey ete Te wwe (0) ele) | ——n +2 — @ ® Figure 9: P-3.1.1 Solution: ] The function f(t) can be obtained from F(w) by doing an inverse Fourier transform, S)= FMP) = 5 [™ Fleyeiat (7s) DS) HO=z [rede win, ha » Ho= LP retttemto aie inl tgiet gy = wigs i c) The sketches for the time functions are given in the figures 10, 11. In the first case, the Fourier transform is real and even resulting in a real, even time function. In the second case, the Fourier transform is pure imaginary and odd resulting in a real, odd time function, Problem 3.1.2] Show that if F( a) F( F(t), then Fo)= f- soar (78) By $5 a5 7 Figure 10: The function f(t) = sin(Wt)/t for W = 1, even symmetry. ea ae eee Figure 11: The function f(t) = {1 — cos(W)]/t for W = 1, odd symmetry. Fels f~ elas ) °) @ se (60) L £. Lf f(t + re" drdwdt = 2m F(0). (81) Solution: a) Fo)= [se at = [toecta= [seas (82) b) Using the triangle inequality, |A + B| < ial + [B|, and considering integration as a limit of a summation, it can be shown that: [toma sf re ilae= f- ls@lat. (83) Note that |e~#*] el wF(w)ei*des| < eo Fenian (84) Note that the same inequality used in part b) was used. 4d) Note that, f - Sltt r)e#Tar = &* F(w) (85) then, we have, ee Slt-+ re F" drdudt = i L Flw)e*dedt anf flt)dt = 2nF(0) (86) ‘Problem 3.2.1] a) Find the Fourier transform for of the raised cosine pulse signal defined by: Lteosmt if -11. (93) c) Using Euler's identity, we get directly an expansion of our function in exponential form as 1 int S(O) = 1 + 008 (nt) = 1+ (2 +e) (94) from which it is evident that Fy =1, Fa = Fi =1/2 and Fy =0 for |n| > 1 Problem 3.2.2] a) Find the spectral density of the real-valued function: _f acxp(-at) t>0 #0 -{ bexp(at) t<0 (95) b) Examine your answer to part a) for the special cases 6 = a and 6 = ~a, particularly with respect to the following chart: fF) Fw) even even real odd odd imaginary 27 Figure 12: Raised cosine pulse Solution:] a) The Fourier Transform can be written as: 7 Oy ela-dult, oe entation, b +a) + ju(b ~ Fo) ft! vonat + [ ac(otieigt = shiek *) (96) b) From the above expression we obtain the following cases: if a=b Fle) = (97) ifa= These conclusions are consistent with the chart given above, with 6 = a corresponding to the first row and 6 = ~a to the second row. Problem 3.2.3] A pulse signal described by f(t) = exp (—alt))rect(¢/T) is repeated periodically with period T. a) Find the exponential Fourier Series beginning with the Fourier transform of exp (—alt|) and the converting to the series. b) Compare your answer with problem 2.7.1. c) Under what conditions can equation 3.15 (equation 3.15 is as follows: Fy, = 7F(nwo)) be used to obtain the Fourier series coefficients ? [Solution:] a) For one pulse , (from table 3.1) (98) 28 Using equation 3.15 2a/T 1 TPC) bene = CREDA (9) b) For T = 2, the result in a) gives: PF) |. (100) But, problem 2.7.1 gives: (101) Relationship (100) is indeed the general term of the Fourier series representation of the periodic function obtained by repeating the function exp(—alt|) while (101) is the general term of the Fourier series representation of the periodic funetion obtained by repeating the funetion f(t) = exp (—alt))rect(t/7) c) For a give function f(t), the formula F, = }F(nu») gives the Fourier coefficients of the periodic signal Df. f(t — KT). ‘These are equal to the Fourier coefficients of f(t) as long as F(t) is zero outside of the time interval 1. In this case, the replica of the original signal, namely F(t— kT), do not have overlap and the resulting periodic signal, namely D~2_,. f(t— kT), is equal to the original signal f(t) over the time interval 7. ‘Problem 3.2.4] The time function f(t) = (1/ov2m)e~t’/?*" (o= constant) is known as the Gaussian function. This function has finite energy and thus is Fourier transformable. Find its Fourier transform. In carrying out your solution it will be helpful to combine exponents, complete the square in the exponent, and then use the finite integral (&, edu = /#. Note that f(t) and F(w) have the same mathematical form; ie., the Gaussian function is its own Fourier transform. Solution: Fw) = Z [Ccesmactertainea, (19 Let u= (t+ jo%w)/V%; then, Fw) = ee"! Sore eee 103) Problem 5.4.2] Show that a more general statement of Parseval’s theorem for for energy signals than Eq. (3.21) is [sor oe= z/ © FW)G"(w)ao. (104) 29 Solution: fs Allowing an interchange in the order of integration, this becomes: £ F()g" (Hae = alt. Fuerte) [ He ddadt. Using Eq. (3.25) [sor of the main text, this becomes: i)dt = tf Lt F(w)G"(2)8(w — 2)dedu = £ F(w)G"(w)dwo. Problem 3.4.3 Evaluate the following definite integrals using Parseval’s theorem: a) [isn 2/z)Pde, b) [ene +2), ) [7 aejtat +2 |Solution:| We know that, a) rect(t/2) <=> 28a(w) then, [rea x [Pin wold resulting in, oe [ [sin w/us]Pdo = b) ol) 55 = |_t “(esd = n/a, a Clee da = 2% [ (eat = x/a, a> 0. °) etl 24 e+e 1? (_2@ \? Qn paiva — AT 4a? /” (sea) dom te fe "Pade = a 30 woortode= [~ [2 [~ reyeas] [E [oc we*e] at. (105) (106) (107) (108) (109) (110) (11) (12) (113) (14) Use the result of problem 3.4.2 to evaluate the following integrals, for a > 0,6 > 0: oe dz 9 eae Sa(be) ‘b) L. 4a?” Solution: a) eval gy eas (118) a de 1 al ae, f-(ossie 7 f Brae aan fo pat ge a= 5 [eam ay (8) b) rect(t/7) = Sa(ur/2) (17) © Sa(bw) 7 * [~ SO y= an f aut (35 3) ae lat = 7 =50-e%). (8) Problem 3.5.1] Use Eq. (3.25) and an interchange in the order of integration to show that: ) FFU = 10. b) FEF} = £9 Solution: 3) Furion= z [ [fo seve *ar] ode us) FUFUOH = z f tte) [dude = i H(r)5(t—7)dr = F(t). (120) ») Feros [~ [fo nme rar] man = J sey [feral ar, 20) FFU} = f. © Flayblet rar = 1-9) (122) [Problem 3.6.1) a) Find F(w) for the f(t) shown in Fig. P-3.6.1 (a), (b). b) Sketch |F()| for r < T for both cases, and compare Solution: Fw) = Are" Sa(wr/2), (123) = Arel*?Sa(wr/2) + Are? Sa(wr/2) = 2A cos (wT) Sa(wr /2) (124) = = q 31 At) Alt) a 7 T © T Figure 13: P-3.6.1 b) Graphs for |F,(w)| and |Fs(w)| are shown if Figures (14) and (15), respectively. Figure 14: The function ASa(wr/2) for A= 1, 7 =1. Prob Use the modulation property to find the function f(t) whose Fourier transform is shown in Fig. P.3.6.2 for the conditions: a) B= A; b) B=-A Solution: a) For A= B, we have, 32 ¢ lly | od ‘y ifs Wyn ) Figure 15: The function 2A cos(wP)Sa(wr/2) for A=1,7=1,T = 2. Fw) Figure 16: P-3.6.2 F(w) = 2Arectw/2W] + A rect{(w ~ wp) /2W] + A rect{(w + w)/2W) Using Table 3.1 and the modulation property of the Fourier transform, fe) = 2a saw) + AW so'sawi) + AZo imtsacwe) = oat +cos wot}Sa(W1). b) Similarly, for A = —B, we have, F(w) = 2A rect[w/2W] — Arect{(w — wo)/2W] — A rect{(w + wo) /2W] and f(t) = 2a sawe) 7 Al sa'sawe) - A eiatsa(wi) 5 oat cos wt}Sa(W) 33, (125) (126) (127) (128) (129) (130) Problem 3.6.4 If f(t) ++ F(w), determine the Fourier transform of a) F2-0), b) #I(t/2) - 1, ©) f(t) cos (t= 1), 4 a) GlFen)- [Solution: a) Assuming € = 2~ t, we obtain, £ f(2— teat = f S(Qe 2 de = eH Fw), b) Assuming € = (¢/2) ~ 1, we obtain, [itera — eae = a 7 eae = DoH? (20), c) Substituting, cos(nt ~ x) = [eit + e-Jlnt-")]/2, we obtain, yom £ SQM Mat Fe [i soeseriat = — 5lFlw—n) + Fw mh Note that e* = e#* = =1. d) We have, fe) =f” P@eag fea) = | P@eag tien) = [Ger @eas rr Sipen)= [~ iwP(w/ryehaa/2 This results in, Sat) = (iw/2)Flw/2) Problem 3.6.5] (131) (132) (133) (134) (135) (136) Find the Fourier transform of the pulse waveform f(t) shown in Fig. P-8.6.5 by differentiating to obtain impulse functions, then writing the transform using the delay and integration properties. [Hint: Consider use of superposition. ] 34 Figure 17: P-3.6.5 Solution: Taking two derivatives of f(t), we obtain f"(t) = (—A/r)é(t +r) + (A/r)4(t — 7) + 246'(2). Therefore, we can write, so ff’ cama +n + (are —miacas +f" aaseae sr which has a Fourier transform —A/r)eit + (Alt) eH" (jw)? (138) ‘Two functions of time, f(t) and g(t), are known to satisfy the following integral equation: a(t) = [ aenrte- Har +50). (139) a) If f(t) = exp(~at)u(t), find 9() b) If F(8) = exp(—alt), find g(t) Computing the Fourier transform of the two sides of our main relationship, we obtain, (140) or = FoR (41) 35, a) From Table 3.1, for f(t) = exp(~at)u(t), we have F( + jw) and, _ _jwta Se) = jote-1-)* fore (142) at) = (+e Mult), a>1. (143) b) From Table 8.1, for f(t) = exp(-—alt}), we have, a (144) wid 2a Brecon apo (145) g(t) = 5(t) + VAN, 4 > 2 (148) Problem Consider the following (volume) integral o - S(u)h(v)é[t— (w+ v)}dudv. (a7) 1) Show that this integral describes the convolution integral, f(t) + h(t). ) Using this integral and the result of (a), show that f(t) « A(t) = A(t) + F(t). c) Use this integral to show that the area under the convolution result of two given functions is equal to the product of the areas under the two functions, d) Repeat part (c) using the inverse Fourier transform of one function in f(t) « h(t) and then interchanging the order of integration. Solution: a) Noting that the impulse function is at v = 1u, the integration over v yields: £ F(w)A(t — u)du = f(t) + A(t). (148) b) We could just as well have chosen the integration in (a) over w f. H(t — vph(v)du = A(t) « F(0) (149) 36 £ LE F(uph(v)s[t ~ (w+ v)]dudvdt = (150) Lf soon f° at (u+v)Jdtdudy = (151) £ £ flu)h(v)dudv = (152) [s0au [™ pone (153) d) We have, ja Flen(e) at= ff seryne—nyarae . i sf. F(u)e"dwh(t—r) drat (154) Assuming € = t — r, we obtain, a Pel Plw)el*"dush(t — 1) dr dt = (185) LE Fw |f h(g)e tag] eM dwdt = (156) [or (w) H(w) Ef do (s7) [POH (e\A(u) do = FO)H(0) (158) Problem 3.7.3 Use frequency convolution to prove the following trigonometric identities (also see Fig. 3.3): a) 2cos* uot = 1+ cos 2wot, b) 2sin? wot = = cos uot, ©) 2eosurt coswat = cos(wi + w2)t + cos(is — w2)t. a) We know that cos(wt) <=> a[5(w — wy) + 5(w +wo)], and, f(t)g(t) > [F(w) * G(w)]/2m, then, 2 cos? uot => 2d {rl6lo— wo) + 6(w 4+ wo)] #x[5(w— wo) +5(o+u0)}} = (159) sf. ‘n(5(u— wo) + 5(u + wo)}n[5(w—u— wy) + 5(w—u-+un)idu = (160) Sle ~ 2un) + 25(w) + 75(w) + x5(e + 2p) — 1+ 608 Qugt (161) b) We know that sin(wyt) <=> —jr[6(w — ap) ~ 5(w + wo)], then, Qsin? wot => 2 Hin lilw ~ tie) = 6(w + wo)] # —Fx[S(w — wo) — 5(w + w0)]} (162) 37 ef. =n{5(u— wo) — 6(u+ wp)]r{S(w — u— ap) — Sw — ut wo) du 8 (w — 2up) + 25(w) + 95(w) — 25(w + Quy) => 1 — cos wut * p{B(u~ uy) + 5(u + o)]a [bw — w~ won) + 5(w ~ w+ w)]du ilu = 0 — 3) + 5+ 04 ~ on) 4] (wy +) + HOw +0 + 02) 5 cos (Wy + w2)t + cos(w — un)t. [Problem 3.8.1 (163) (164) (165) (166) (167) (168) ‘Sketch the results of the following convolution operations (where fo > 0 in all cases). Check your result by writing a Fourier transform, of each function, multiplying and writing the corresponding time function. a) Ad(t) * BS(t ~ ta) b) Ad(t-+ to) + Blt — to); c) Ad(t — t)) » BS(t — to); 4) A[B(t-+ to) + 4( to] » BIS (E+ to) + 5(¢ ~ to))s e) A[d(t +t) + d(t — th] + BS(t + to) +4(t— to)], to > tas ‘Solution: Recall that convolution of two functions f(t) and g(t) is given by: Noa) = [> serate drs Using this, the convolution of our given functions becomes: ° AS(t) + BA(t — to) = £ Ab(r)B6(t— 7 — to)dr = ABS(t — to); F{AS(t)} = F{BS(t- te) = Ba +; The product is ABe~*“", and the inverse Fourier transform of the product is, FY ABe™i%} = ABO(t — to). which agrees with our equation obtained earlier. b) AB(t-+ to) # BO(t— to) = ° A8(1 + to) BO(t — 7 — to)dr = ABS(t); 38 FASE + to)} = Ach"; F{BE(t— to)} = Benito; ‘The inverse Fourier transform of their product is given by: Fo Aei# Berd} = ABE(t) Which agrees with our previous result, °) AS(t ~ th) # Bé(t to £ Aé(r ~ t1)BA(t — 7 ~ to)dr = ABO(t — to ~ tr); F{AS(t — y)} = Ae"; F{BA(t — to)} = Be; ‘The inverse Fourier transform of the product is given by: Fo" Avi" Be} = ABE(t — to ~ th) Which agrees with our previous result, d) ‘The convolution expression given is equal to: [ A[S(r + to) + 47 — to)]B(S(t — + to) + 5(t — 7 —to)]dr = AB[S(t + 2to) + 26(2) + 5(t — 2to)]s By evaluating the Fourier transform of the individual terms, multiplying and finding the inverse Fourier transform, as done in parts a)—c), it can be found that: Ff Aledo 4 e-F4t0] Blois + estoy} = ABLS(t + 2to) + 25(t) + 5(t — 2to)] which agrees with our previous result. ¢) The convolution of the given expression is equal to: [Alb + 2) + 8(r = eI BI(t— 740) + (E17 toe AB[S(t + th + to) + 4(t ~ th + to) + 5(E + th — to) + 5(¢ — tr — to) Likewise , F'{Aleitt 46° 5ett ) Beit +e F4te]} — AB[S(t-+ti +o) +4(t+ti to) +6(t-th +to)+5(t—t1 ~to)}; Which also agrees with the above result. Wvaluate the following convolution integrals; check your result by taking the Fourier transform of each function , multiplying, and finding the inverse Fourier transform. a) u(t) *etu(t); b) evttu(t) + e-Mu(t) c) eA cosupt 39 u(t) xe-tu(t) = £ Je u(t - nar = far ‘The integral is zero if t < 0 and is equal to (1 — e~*) if t > 0. Then, we have, u(t) «etu(t) = (1—e“)u(t) The product of their Fourier transforms is given by: 1 nite 1 rile) 2 1 [e+75)] [Aa] joGotty * Jott ~ jo josi tO) The Inverse Fourier transform of the above expression is given by: Je Mult — r)dr tnd te~**u(t) a=6 [(e* — er) (adult) a Ad ¢) Following the same procedure, it is seen that: ell wcosunt = £ e-l"l cos(wor)dr 2 en ft €°7 cos(wor)dr + et je cos(wor)dr 2a pag one ‘The Product of their Fourier transforms is given as: [zeal {r8(w — wp) + 74 (w-+ wo] 7 [r5(w — wo) + 78(w +u0)] ‘The Inverse Fourier transform of this expression is given as. 3 coswot, Fup ‘This is also consistent with the earlier result, 40 Evaluate f,(t) * fo(t), fa(E) * fa(t), and f2(t) * fa(t) for functions defined by waveforms shown in figure P- 3.8.3 he He) Be 1 1 1 os: ° hr 8 Figure 18: P-3.8.3, fe shift fy with respect to fr, this results in, A@)* ht) = o [ff 00042) 467+ 1) 0509) 48571) +89 — 2M ever Problem 3.8.7] Two functions of time, f(t) and g(t) , are defined by: ro-{5" octct, 0 elsewhere emt o 0? Solutio: a) 1/(R:C) H(w) oT (> jo4 T/C) where Ry = (Ri R2)/(Ri + Re) Using the relationship between the input and output power spectral densities of a linear system we get the result: L/(Rsey? FER CP Svol a 2 b.) Using table of Fourier transforms, we get, Rog(t) = c.) The rms value of the output is given by: BO = Ra.(0) = Ae: “ Integrating S,,(w) results in the same value. .) As C+ 0, H(w) becomes a resistive divider and because the input source is assumed to be white, the mean square output > oo. 50 Problem 4.4.1] a.) Determine the power spectral density of F, exp (jwot) by first finding the autocorrelation function, then taking the Fourier ‘Transform of the autocorrelation function. b,) Repeat part a.) for [F, exp (jwot) + Fh exp (j2wot)]. c.) Extend your result in part (b) to S$2._o. Fem Tia anak [ME memindoria= inte Syle) = F{Ry(2)} = 2x] /?5(e — 0) a). ny) = BP tetemte emma + meme = |Rife" + [Fe iFrom this we get: Sp(w) = 2m|F,|?5(w — wo) + 2m] F2|75(w — 2wo) c.) The cross-terms integrate to zero and we get: Ry(r) = D2. [Fale The power spectral density is then found to be given by: S4(w) = 20 S> |Fa)?6(w — maw). [Problem ‘The complex pulse signal ro={ eet Otc th 0 ast 2t,. a.) Find the autocorrelation function Ry(7), and sketch the real part. b,) Find and sketch the power spectral density $j(w). Solution: a.) We want to compute the integral, 1 7 R= Ff romesna We select the range of the integration as [0,7]. We have, . eset Stet t) = 0 ysecT 51 Note that for r < 0, the function f(t +) is obtained by shifting f(t) towards positive t axis and vice versa. For —t <1 <0, we have, o O th. Similarly, for 0 < 7 < th, we have, ft+7) dial) O + *B)/e]— tan“"{(wo —eB)/e}}. (174) Note that, J oreate = ta (2). (175) ‘The desired ratio is: So an Ny 7 tas ay BY toa Yow = 9B on Problem 4.6.1, Determine the autocorrelation function of each of the following pulse waveforms by first tak- ing a Fourier transform, next taking the magnitude squared, and then taking an inverse Fourier transform. a) e-**u(t), b) rect(t/ti), ©) Sa(W) Si " a) . 1 1 1 al ano jose Frat — 2" el, (177) ’) rect(t/t1) <=> t1Sa(wt; /2) => t2Sa" (wt, /2) <=> t)A(r/ty). (178) °) Sa(Wt) = apreetio/ 2W)] > (i) seer = W5alvr). (179) 53 Two pulse waveforms 2(t) and y(t) are shown in Fig. P-4.6.2. Determine and sketch ra(: ry(7), and rey(7). x(t) y(t) Figure 20: P-4.6.2 [Solutions] ‘The autocorrelation functions below are zero outside of the intervals given. a) te(t) = [ian +7, for -1<7<0, (180) = [ve ining the results, we have: 1 for 0 2 b) For y=2 Ri -2R,Ry + RE=0=> Ry = Re. (189) je frequency transfer function for the RC bandpass filter shown in Fig. P-4.7.2 can be approximated by that for two independent cascaded RC filter sections if Ri < Rz. Using this approximation, determine an expression for the spectral density of the thermal noise voltage at the output terminals of the filter. pW 0 CG RL Ry v(t) Figure 21: P-4.7.2 (Solution: To compute the effect of the noise, we assume that v; is short circuit. In this case, as Ry > Ry, we can assume that the parallel combination of 2; and Cy is short circuit and Re and Cy are in parallel, then, the transfer function from the noise source of Rs to the output terminal is equal to, Wier © Rot (1/jCw) ‘This results in the following power spectral density for the noise at output. A( KT Rs|H(w)|?. 55 Problem 4.7.3 Find the rms thermal noise arising from the resistance bridge circuit in Fig. P-4.7.3 under the following limiting conditions: a) Ro 0, b) Ro + 0. vi(t) Lee a Figure 22: P-4.7.3, Solution: We compute the output resistance referred to the output terminals. ‘This results in, 5) Req = (Ril R)+ (RI R)=R (190) Req = (R+R) || (R+R)=R (191) In both cases, assuming a bandwidth of B, we get: \/v2(t) = VARTR.B = VIRTRB. Note that if we proceed using the superposition principle, in both cases of (a) and (b), consid- ering the division of voltage in the corresponding circuit, the effect of each of the four noise sources is multiplied by a factor of (1/4) when it appears at the output terminals. Then, adding up the effect of the four sources, we get the same result. ‘Problem 4 ‘The input of a voltage amplifier is connected to a1 — K9 resistor and the output to a 100-K2 resistor as shown in Fig. P-4.7.4. The voltage gain of the amplifier is 10, the input impedance is 1MB® (assumed noise-free), and the output impedance is 1000 (noise-free). The bandwidth of the amplifier is 1MHz and the amplifier noise is 10V rms, referred to the input. ‘The temperature of the entire system is 300K. compute the rms thermal noise at the output (point 2). 56 Figure 23: P-4.7.4 Solution: Output noise arising from thermal noise in input circuitry: ‘7 (t) = 4&7 B(1k)(1000/1001)*(10)(1000/1001)? = 1.65 x 107 v2, (192) where K = 1.38 x 10-9 and K = 300. Note that the two (1000/1001)? terms arise from the voltage dividing at the input and output of the amplifier and (10)? is the power gain of the amplifier. Output noise resulting from amplifier noise: ‘n§(t) = (104V)?(10)?(1000/1001)? = 9.98 x 10-° V? (193) Output noise arising from thermal noise in output circuitry: ng(f) = 4kTB(100kM)(1/1001)? = 1.65 x 10-* V7, (194) nz(e) ni (t) + n3(6) + n3(t) = 9.9817 x 10°? V? (195) nat) = 99.91uV. (196) Problem 4. ‘A sinusoidal generator develops the waveform vi(t) = A cos wot as the input to the RC filter shown in fig. P-4.2.4, a) Derive an equation for the rms value of the thermal noise at the output of the filter. b) Determine the value of C’ which will yield the highest S/N ratio (ie., ratio of average signal power average noise power) at the output. Solution: ‘The frequency transfer function is: 1/(RiC) jo+1/(R,cy “he Br RRs Hl) Ri + Re (197) 87 Ry c Re volt) wl C Figure 24: P-4.2.4 ‘a) Using circuit analysis: AMR) oo {wot — tan“ (woRpC)), (198) af) = ob + 1/(BCP The rms value of the output is: 1 AMC) ave) v8 og + 1/(RyCP Then, R; and Rz become parallel. In yna(d) = VETTE. To compute the effect of the noise, we short circuit this case, referring to example 4.7.2 (page 193) of the text, we obtain The overall result is, _A/RC) (200) JERTIC u8 + 1/(RpC)? Setting the derivative of $/N with respect to C equal to zero gives the optimum C. S/N = Problem 4.7.7 Compute the noise equivalent bandwidth of the RC bandpass filter shown in Fig. P-4.7.2 for RyC; = RzC2, and using the approximation described in Problem 4.7.2 for Ry < Rz. Choose the midband frequency to be that frequency at which |H(w)| is maximum. Solution: For the conditions specified, we can write: jwRC Gores (201) and therefore (202) Setting d|H(w)|?/dw = 0 and solving for wy, we get: wo = 1/(RC) 58 Using Eq. (4.67) of the text and noting that |H (wo = 1/RC)|? = 1/4, we obtain: (wR)? 1 a mal | [ercy rip™ = are (203) Problem 4.7.8] ‘A bandpass amplifier is to be designed to meet the following specifications: By = 200kHa (S/N output 2 20 4B Max. output = 1 V rms across 300 @ Estimated noise temperature of input stage = 1000 K , in dB, for which you would design the amplifier to meet these What is the maximum gai specifications? Solution: On the basis of the output S/N > 20 dB or 100 and the maximum output, we can find the ‘output noise power as So = 100N (204) SotN = v/R (205) 1o0N+N = (1)°/300 + N=3.30x10-* W. (208) Without any additional input thermal noise, the effect of the noise of the input stage at the output is equal to, N = kT-ByG». So that: x 10-8 G, < 80x10 - Oey < ER = 1196 10 + G, < 100.848. (207) [If T) = 290 K were assumed at the input, this would change the answer only slightly to 99.7 dB). A certain amplifier has input and output resistances of 509 and a noise-equivalent bandwidth of 140kHz. When connected to a matched source and a matched load, the net gain is 50dB. When 50M resistor at 290K is connected to the input, the output rms noise voltage across 502 is 100,V. Determine the equivalent noise temperature, T., of the amplifier. Solution: Pout = (100 pV)? /50 = KT>BGy + kT-BNGy (208) 59 eu Res | | (100 pV)?/50_ EBNG, from which Ty = 1035 — 290 = 745 K. (209) ‘A General Problem on Noise: | Consider the following circuit where, v4(t) = cos(wot + ¢) and vali) find the power spectral density of the signal and the noise at the output terminal, Solution: We know that the power spectral density of a periodic signal with the Fourier series expansion Dy Freinsot is equal to $4(w) = 20D, |Fal26(w — nwo). We have, rectr(t/r), Tz on (0) = 3 [estore 4 ersten] So Ryemet ‘This means that, Rae, Fase #2, Fy=0,n#1,-1 As |Fil? = |[F-al? = (1/4), then, Similarly, for vz, we have, Fy = (1/2)Sa(nx/2) and, Sule) (7/2) 7 S0?(nn/2)6(w ~ nwo) ‘The transfer function from vy(t) to output is equal to (short circuit v2), Rojo Ry + jw Trl), Ballo (m+ a) +R tile 60 ‘The power +r spectrum of the output signal due to v; is equal to, [Aa(o) 7S. ( (2/7) [Eh (w)[?6(w — wo) + (2/7) Hi(—wo)8(w + wo) Due to the symmetry of Hy(w), we have, Haw) /?Suy( (2/7)| Hi (wo) /?(5(w — wp) + 5(w + wo) The transfer function from v(t) to output is equal to (short circuit v1), tol (@+ 325) Hy(w) = es [oneyii (B+ 3S5)) ‘The power spectrum of the output signal due to vs is equal to, |Ha(w)?Se, (0 ir /2) > |Ha( ruse) |*Sa2(nx/2)6(w ~ m2). As the noise sources are in series with the resistors, their effects can be computed using the same transfer functions. This means that, to compute the effect of the two noise sources at the output, we should replace S., by Sm, to compute the signal power at the output. ‘Problem 5. band suppressed carrier modulator operating at a carrier frequency of 200Hs, 2KTR, and Sy, by Sy, = 2KT Rs in the relationships used A modulating signal f(t) [with Fourier transform F(w)] is applied to a double Sketch the spectral density of the resulting DSB -SC waveform, identifying the upper and lower sidebands, for each of the following cases. a.) f(t) = cos 100nt b) Solution: a) This gives the b) { [1+ cos (w/200)]/2 Jw] < 2007 0 elsewhere g(t) = cos 100ntcos 400nt = 1 cos 300nt + 5oos 500mt corresponding Fourier Transform: G(w) = F{8lw — 8007) + dw + 800n) + 5(w — 5007) + 5(w + 5007)] 1 5 P(e 400m) + 5 F(w + 4007); 6 S00 3008 soos 5008 oe 400-2008 200m 400 Problem 5.1.3] We wish to examine the frequency and phase response of one low -frequency oscillator relative to the second. We decide to multiply the outputs of the oscillators to make the comparison. However, as a result of difficulties in building a low frequency multiplier we instead choose to use the DSB-SC modulators and bandpass multiplier, as shown in Fig. P-5.1.3 a.) Determine the expression for the DSB-SC signals. b,) Determine the expression for g(t). ¢.) Sketch a magnitude Spectrum of g(t). 4.) Under what conditions does g(t) represent the desired product of the two input waveforms? cos gt Go £05 [(6oq + Soy + 8 Solution: Fi(t) = cos(wnt) c05(Wet) £2(t) = cos[(wm + Aw)t + 6] cos(wict) You can simplify /expand these if you wish, b) alt) = y(t) x $2(t) = Fi [cos (Awt + 0) + c08 (2umt + Awt + 0)] [1 + cos 2a]; 62 c.) We know that, cos(Awt + 6) + cos(2unt + Awt +6) has the Fourier transform, 75 (w — Aw) + me 5(w + Aw) + me 5(w — urn — Aw) + we~295(wW + 2m + Aw) Let us show this Fourier transform by F(w), then we have, |F() rw — Aw) + 15(w + Aw) + 15 (w— 2wm — Aw) + 16(W + 2m + Meo) Note that here we have simply added the magnitudes of different terms because we have impulses. However, in general, this is not possible. ‘The resulting magnitude spectrum after multiplication by (1/4)[1 + cos 2uet] is equal to, 1 1 l F(w)| + a1 (we ~ Qu w GIP) + GPW — e+ GIP w+ 200)] d.) g(t) will represent the product of the two waveform if 2u. > (4um + 2Aw), or we > (2uim + Aw), and a low pass filter is used at the output. Problem 5.1.4] A sinusoidal signal f(t) = cos2000rt is multiplied by a periodic symmetric tri- angular waveform (cf. Table 2.1) with unit peak amplitude and 7’ = 100psec. ‘The output of the multiplier is applied to a low pass filter with a unity gain within the passband, a.) Determine the minimum and maximum bandwidth of the LPF if the output is to be a DSB-SC waveform corresponding to f(t) b,) Determine an expression for the output of the LPF under the above conditions c.) Can this system be represented by the DSB-SC modulator shown in Fig. 5.1.(a) under the above conditions ? If your answer is yes, the determine the two inputs to the modulator. fa.) The signal has a bandwidth of 1 KHz. For the triangular waveform fo = 1/T = 10 KHz and it has odd harmonics only. Therefore, the LPF bandwidth must satisfy the condition 1LKHz < B < 29KHz where 11 = 10 (first harmonic) + 1 (bandwidth of the signal) and 29 = 30 (third harmonics) ~ 1 (bandwidth of the signal). Use sketches to visualize this properly. b.) Using Table 2.1, we represent the triangular signal by the corresponding Fourier series. The coefficients for the first harmonic are equal to (2/n)?. and LPF, we have: 19 = [Glee Glome borden] = (2/n)?cos22000nt + (2/)*cos 18000nt Assuming unity scale factor for the multiplier 63 Note that wo = 20000m and w: = 20007. c.) Yes, the equivalent inputs are: cos 2000rt and (2/1)? cos 20000mt. Problem 5.1.6] When the input to a given audio amplifier is (4cos800nt + cos 10004) mV, the measured frequency component at 1000Hz is 1V and the ratio of the frequency component. at 500H to that at 1000Hz is 0.002. Represent the amplifier output-input characteristic by eo(t) = arer(t) + aales(t)]?. a.) Bvaluate the numerical values of a1, az from the test data given. (This type of test is known as the intermodulation distortion test.) b.) What would you expect to be magnitudes of the frequency component at 800Hz and at 1600H2? 64 Solution: a) €olt) = a1(4cos800nt + cos 10001) + a2(4.cos 800mt + cos 1000nt)? = 4a cos 800rt + a; cos 1000R¢ + 8.5a2 + 8a cos 1600nt + 0.5az cos 2000mt + 4a, cos 200mt + daz cos 1800nt For the given values 0.5a, b) @400H2 : 4a, mV @800Hs : 8a, mV 1000 mY, then a, = 2000, and 2a; /az = 0.002, a, = 2 Problem 5.1.10] Two measurement systems are shown in Figure P-5.1.10. Find the output of each system if e;(t) = coswit. What type of modulation is present in each and how should it be detected ? f0, NR =a wn) apLaan & 7 l rout | LO, Ry= RU + afl) Liz = ALL + xt) aif << Las = kL - ax(t)) ‘Solution: a.) It can be shown that If we substitute the relation we obtain the relationship 65 Given |af(t)] <2, this can be simplified to give ea(t) & =2 f(t) coswit b.) The voltage developed across the inductances is equal to, eo(t) e1a(t) ~ zat) AL + ae(t)] coswt — [1 — ax (t)] coswit = Yake(t) cosurt Both a.) and b.) are examples of DSB-SC modulation. They require synchronous detection. Problem 5.2.4] For the sinusoidally modulated DSB-LC waveform shown in figure P-5.2.4 a.) Find the modulation index. b.) Write an expression for the waveform in the form of equation (5.18). ©.) Sketch a line spectrum for the waveform. 4.) Show that the sum of the two sideband lines in part c.) , divided by the carrier line yields the modulation index. ¢.) Determine the amplitude and phase of the additional carrier which must be added to make the waveform shown to attain a modulation index of 20% £.) Repeat part e.) to attain a modulation index of 80%. 125 235 135 (Solution: a.) Consider the AM signal 9(t) = A(1+mcoswmt) coswet. Let (t) = A(1-+mcosut) denote the corresponding envelope. We have Yinax/ min = (1+ m)/(1—m), or, m= Pinas = Ynin VPmaz + Vinin In this case, we obtain, 125-25 _ 100 125 +25 ~ 150 b.) We have Yinax = A(1 +m) = 125 resulting in A = 75. (The same result is obtained by using ‘min = A(L—_m) = 25.) Then, m = 66.7% A(t) = 751 + 0.687 coswynt} coswet 66 c.) Straightforward. 4.) Each spectrum line of the carrier has amplitude 75/2 tude 75 x 0.667/4 = 12.5, then, (12.5 + 12.5)/37.5 = 25/37.5 e) 7.5 and each sideband has ampli- 6.7% O(t) = (75+ A) cosuct + 50 cosuimt cosuict; This gives the value of m as m = 50/(75+ A) = 0.20 resulting in, A = 175 V. f.) Using the same procedure as above we get: 50/(75+ A) = 0.80; Solving for a yields A = ~12.5V =12.5e~#* V Problem 5.2.6] A given AM (DSB-LC) transmitter develops an unmodulated power output of KW? across a 50-ohm resistive load . When a sinusoidal test tone with a peak amplitude of 5V is applied to the input of the modulator, it is found to the spectral line for each sideband in the magnitude spectrum for the output is 40% of the carrier line. Determine the following quantities in the output signal: a.) The modulation index b.) The peak amplitude of the lower sideband. c.) The ratio of the total sideband power to carrier power, d.) The total power output. ¢.) The total average power in the output if the peak amplitude of the modulation sinusoid is reduced to 4.0V. Solution: ‘a.) Using the result of problem 5.2.4(4), we conclude that the modulation index is equal to m= 0.8. This results in the following modulated signal, A(L +.0.8 cos wyat) coswict = A coswet + 0.44.c08 (we ~ Wm)t + 0.44 COS (We + Wim )t b,) 42/2 = PR => A= V/(2)(1000)(50) = 316.22V => Peak amplitude of sideband = 0.44 = 1265. c.) P,P = m?/2 = (0.8)?/2 = 0.32. d.) Using the equation: (2 Fy @D/R = (1/ RA + (3) (3) ma? and replacing A?/(2R) = 1000, We get the result : P = 1000 x (1+ (m?/2)] = 1.32kW 3PCC power ratings for AM broadcast transmitters are for an average carver (Ie. unmodulated) power. 67. ©.) Pe = 1000[1 + (4/5)%(m?/2)] = 1.025kW Problem 5.2.10] Let f(t) = cos wyt in Eq. (5.28) and add the term age*(t) to Eq. (5.27); then revise Eq. (5.29) and derive an expression for the modulation index m. i(t) = aye(t) + aze?(t) + ase*(t), (210) where e(t) = €08 wnt + 008 wet (211) Expanding and collecting terms at the carrier frequency, we have volt) = kRlay + a5(3+ b)/2] cos wet + 2bRa2 C08 Wat C08 wet + (kRas/2) C08 2wmt cos wet (212) ‘The third term in the expression represents second-harmonic distortion in the output. Defining ‘the modulation index for the fundamental, 4az m3, + (8+ Pag (18) ‘Thus it is desirable to keep as very small, and not let k become large. Problem 5.2.11 A sinusoidally modulated DSB-LC waveform #(t) [ef. Eq. (5.18)] is applied to a square-law device, such that the output voltage ¢o(t) is eo(t) = [f(t)]?. Show that the ratio of the second harmonic to the first harmonic in ¢o(t) is equal to m/4. Solution: eo(t) A*(1+ moos wnt)? cos* wet, (214) colt) = $A%(L+ 2.08 unt + my C08 2uimt)(1 + 2c08 wet) (215) 2 ‘The ratio of the second harmonic to the fundamental is: m3/2_ 2m Problem 5.4.2} ‘A SSB—LO, transmitter is modulated with the input f(t) = cos 2000rt. In complexed-valued notation, the output waveform can be written as Bssp_zo, (t) = R{A[1 + mf(t) + jmf(t)]e* (216) 68 a) Evaluate f(t). b) What is the maximum value of m if envelope detection is used? ¢) Determine the numerical values of A, m if the unmodulated power across 50 ohms is 500 W, and the average power is found to increase 50 4) Find the peak envelope power (PEP; see Problem 5. the modulated condition in part (c) 1) under both the unmodulated and Solution: a) S(t) = cos (2000nt — 1/2) = sin 2000nt [ef. Fig. 5.31) (217) b) The envelope is: B= [OF mcos wit)? (min gt)? (a8) = VII 1 + PR os wnt @) Using a binomial series approximation, we obtain, a(t) Vit me (1+ mY 608 wnt itm ‘The power of the cost term is equal to 0.5m?/(1+ m?). ‘To compute the proper value of m, we know that the total power of r(t) is equal to, 1 +m? (AC power m?) and we set the criterion of having THD < 0.1. °) P(t) = Acos wet + mAcos (we + mye (220) A? mia? R= Ate e ee (221) from which: A = 223.6V, m = 0.707. d) Unmodulated: pep =“. =s00w, 222) pp = A. = s00W, c Modulated at m = 0.707: (140.707)24? PEP= a = 1457 W (223) Problem 5.5.1 6 A vestigial sideband signal is generated from an input f(t) by first generating a DSB-LC signal (m = 0.8) with a cartier frequency f, = 10 kHz and then passing this signal through a filter whose magnitude frequency transfer function is 1+ c08[(w ~ w)/8000], Jw ~ wo] < 80007 (224) 0 elsewhere (ay ¢0)= { where wy = 24,000n. Find an expression for the resulting VSB signal, sketch the spectral density, and calculate the peak envelope power (cf. Problem 5.2.3) to average power if a) f(t) = cos 1000nt. b) f(t) = cos 2000zt. c) f(t) = cos 4000r¢. The general solution is 9(t) = [(1 + 1.05 Wat) cos wet] @ h(t) (225) with the Fourier transform, Fv) x {5S(e ewe) + PS (we bi) + Bw (we — mI} HU (226) ) We = 200007, wm = 1000r. Gt) = cos wet + 0.4[1 + cos (3m /8)] C08 (We + Wn )t + 0.4[1 + cos (5%/8)] COS (Ww, — wn )€227) = €08 wet + 0.558 c0s(ue + ua) + 0.247 c08 (se ~ Wn )t (228) PEP __(140.853-40.247)? , iP = 1 (OlbsS)7r (U:247) eae (229) b) we = 200007, wn = 200m. A(t) = cos wet + 0.41 + cos (20/8)] C05 (we + Win) t + 0.4[1 + c08 (62 /8)] €08 (we — sern)4280) = cos Wet + 0.688 co5(We + Wm)t + 0.117 €08 (We ~ wn) (231) (1+0.683+0.17)? _ . Prog .683)?-+ (Oli ee ©) We = 200007, wn = 40007. O(t) = cos wet + 0.8 c08 (we + Wen )t (233) PEP _ (1+0.8)? _ Pavg 1+ (0.8)? ~ 1.98 (234) 70 A noise waveform with a power spectral density S,(w) = 10-Tevl+l/* W/Hz, where a = 2m x 107/sec, is passed through an ideal BPF with unity gain, unity resistance levels, and a bandwidth of 200 kHz centered at 10 MHz, a) Determine the mean-square values of the in-phase and quadrature components of the band- pass time representation given in Eq. (5.59). b) Repeat part (a) assuming that S,(w)is constant (at the center frequency value) across the bandwidth of the filter. c) Repeat parts (a) and (b) for a bandwidth of 2 MHz. Let W be the low-pass bandwidth [i.e., 100 kHz for (a) and (b)| a) Lowpass approach: using Eq. (5.05), we have Spelt) = Sng(w) = 10-Tfer Heals + er beteol/ah, Jef teil Ke 1 eW/ Wie vanla [Tel 4. el dy = 2e7 "e/a — eWla ersnle f fetta ewe t 1 = ee?! — 1] — 9.014715 V? Bandpass approach: A 0 7 7 ‘ wD = if 9! dun = De He! — e-1] — 0.014715 V’ w b) Lowpass approach: Ww gyi tele [ade = ro 0.14715 V? oe Lw * 2 HW AD = 5-10" 7 e-¥! dw = 0.04e~* = 0.014715 V?. For constant $n(w) nB(E) = nF) = 0.4e7* = 0.1472 V2 1 (235) (236) (237) (238) (239) (240) (241) (242) Problem 5.7.1 ‘A DSB-SC and a SSB-SC transmission are each sent at 1 MHz in the presence of additive noise. ‘The modulating signal in each case is band-limited to 3 kHz. The received signal power in each case is 0.2 mW. The received noise is assumed to be white with a (two-sided) power spectral density of 10-* wW/Hz. The receiver consists of a band-pass filter whose bandwidth matches the bandwidth of each transmission, followed by a synchronous detector. a) Compare the signal-to-noise ratios at the detector inputs. b) Compare the signal-to-noise ratios at the detector outputs c) Repeat part (a) if the (two-sided) power spectral density were 10°/|f| #W/Hz. Would a “white noise” assumption be valid here? 0.2mW. Wao aW aye Tay = 107 fr DSB, (243) 0.2mW = ago > aW/Hay(@eH = 3*9f S8B, (244) b) From Eqs. (5.73) and (5.80) So Si Wr = 2p 7933 for DsB, (245) = 33.3 for SSB, (246) Thus the net S,/N, ratio is the same for both systems in the presence of additive white noise. ) For DSB fo43000 193 M “Z-af = 2000(In(10° + 10%) — In(10® — 10°)] = 12 ¢W (247) For SSB,., we have: Noe i ‘50 10° 7 2000[In(10° + 10%) — In(10°)] = 5.99 pW (248) For SSB_, we have: fe 10° : 5 — 198) df = 2000fIn(10°) ~ in(10° — 10%)] = 6.009 xW (249) N=2 Iy.-s000 f ‘Thus, the white noise assumption is good here as a result of narrow bandwidth, even though the noise power spectral density is not flat, 2 roblem 5. a [Problem 5.7.2] Show that, strictly on an average power basis, use of an envelope detector in a DSB-LC system results in a linear (S/N), versus (S/N); selationship. What makes this relationship nonlinear for commercial AM systems? For the DSB-LC signal, we have, (Eq. 5.81): Si(t) + ni(t) = [A+ JO) coswct + ne(t) coswet —n,(t) sinwet (250) Assuming the signal and noise to be zero-mean, i.e. F(t) ie(t) ° (251) u(t and that the cross-correlations with the noise terms are zero, i.e., FOne(8) = nelt\n.H = Fn.() =0 (252) we get, HO? = 5 [4° + PO + nF) + BO] (253) Because the signal and noise terms are additive in this result, the S/N dependence is a linear one in terms of average power. However, in AM detection the envelope is taken first to demodulate the signal, then the power is computed. In this case there is no longer a linear relation, as exhibited by Eq. (5.91). (Note that the subject of noise in AM using envelope detection, and in specific Eq. (5.91), has not been covered in the lectures and is not included in the exam.) Problem 5.7.4 ‘The DSB-LC signal (0) = 3cos(10, 000xt) + cos (L000zt) cos (10,0004) V (254) is present with additive white noise whose (two-sided) power spectral density is 1 4 W/Hz. This signal-plus-noise is passed through an ideal low-pass filter with a bandwidth of 10 kHz, Assume all resistance levels are 1 ohm. a) Compute the average S/N ratio at the output of the low-pass filter b) A synchronous detector is used to demodulate the above signal. Compute the average S/N ratio if the output of the detector is filtered to 0 < f <1 kHz. ©) An ideal square-law detector (j.e., the signal-plus-noise is multiplied by itself) is used to demodulate the above signal. Compute the average S/N ratio if the output of the detector is filtered to 0 < f < 1 kHa, [Hint: Use frequency convolution, and signal-signal, signal-noise, noise-noise products. 3 Solution: a) Ng = 2%(n/2)B= 2x 10-%(108) = 0.02 W (255) Si = PH) = (8)7(1/2) + (1/2)(1/2) = 4.75 W (256) z = 237.5 (23.84B) (257) b) After LPF eo(t) = (1/2) cos(1000n2) (258) 5S. = = (1/2)°(1/2) = 1/8 (259) (260) Using Eq. (5.72), No = Nj/4 = 0.005 and S./No (14aB). Note that using Eq. (5.88) for m= 1/3 (which is the modulation index for the signal given in 254, gives (2/19)(287.5) = 25, checking this result. c) After LPF colt) = {l6(t) + ne(0) coset + n,(@)sinwt?}, (261) 3.cOS wit + 3re(t) + Me(t) COS Wat + 7 082g + 5720 + FO) (262) The first term in this result is the modulated signal, with the average power (3)2/2, the second (3)7(0.02), the ratio of these two terms is 25, the same as was obtained in (b) using synchronous detection. The third term represents the term is the primary noise term, with average power (3)°Ni harmonic distortion, with average power (1/4)?(1/2). ‘The forth term represents a bandpass in phase noise term around frequency wm. This noise term has the power N;/2 = 0.01. The last two terms represent noise-noise interactions in the square-law detection. ‘To compute the power of these terms, we neen to know the probability density of the noise. We assume that this is a Gaussian distribution. For a Gaussian distribution, we have n? = 3n? = 3 x 0.02. ‘Taking the effect of the (1/2) factor into account, the power of each these two noise terms is equal to 0.03/2. The overall result is, So (3)?/2 ae aR eae: iors = 23 (188-48) (263) Note that the noise-noise term do not contribute appreciably here because the input S/N ratio is relatively high. Problem 6.1. Determine the instantaneous frequency, in Hertz, of each of the following waveforms. a) 10 cos(1007t + 1/3) 74 b) 10.cos(200n¢ + 10 sin xt) c) 2exp[j200rt(1 + y7)] 4) cos 200mt cos(5 sin 2rt) + sin 2007 sin(5 sin 2xt) Solution:] The instantaneous frequency, in Hertz, is where 4 is the instantaneous phase of the signal. a) id 1007 Jp qlldort+ 2/3) = = 50 He b) HG 200Rt + 10sinnt) = 100 + 5coszt He °) 1d Fe gil200H(1 + vi)] = 100 + 150V Ha a) cos 200rt cos(5 sin 2nt) + sin 200n¢ sin(5 sin 2) = cos(200nt — 5 sin 2rt) 1d, ne Si. = go y(200nt — sin 2nt) = 100 ~ Scos2nt He Problem 6.1.3 (264) (265) (266) (267) (268) (269) !) Find an approximation to the Fourier series expansion of the angle-modulated waveform U(O) = R[Aexp(juct) exp(jAsinwmt)] for small # by using the MacLaurin series expansion for exp(z) and retaining only the first two terms in the expansion. 'b) Sketch the line spectrum of the approximation to ¢(t), as determined in part (a). ) Determine the Fourier transform (spectral density) of the approximation to Y(t), as deter- mined in part (a). Solution: exp(j sinumt) © 1+ jBsinumt = 1+ Beant - Berient (270) LA exp(ju-t) exp(jAsinwynt)] ~ Acosiastt AP cos(ueHum)t AP €08(.—Wm)t = Acoswt—BAsinvmtsinwet b) Easy. °) AB(w se we) + FAP Su & (ue +n) - TE sus (we = Wm)} (71) Problem 6.2.1 ‘A 1-GHz carrier is frequency-modulated by a ten-KHz sinusoid so that the peak frequency deviation is 100 Ha. Determine a) the approximate bandwidth of the FM signal; b) the bandwidth if the modulating signal amplitude were doubled; ©) the bandwidth if the modulating signal frequency were doubled; d) the bandwidth ifboth the amplitude and the frequency of the modulating signal were doubled. Solution: a) a Be 2 e001 + NBFM (272) BS 2%fy = 20 KHz (273) b) 2 A Be = BP = 002 + NBFM (274) Box 2% =20 KHe (275) °) Aw 107 Bo xe (are) BOX 2m =40 KHz (277) 4) B= Be 8 De = 001 + NBFM (278) B® 2fm= 40 KHz (279) 76 Problem 6.2.3 ‘The upper sideband of an AM waveform (DSB-LC) with sinusoidal modulation and modulation index m is multiplied by a factor «, where: 0 < a < 1. Derive a relation for the peak (i.e., maximum) phase deviation from the carrier as a function of m and a. ‘Solution: amA 2 dt) = Acoswet + A oo (ue ~ Wm) t + NBFM. Then, we have, eile Iocoswet + Jr cos( ve + Wm)t — Jr 608(we — wm) (283) We make use of the identity, 2cos Acos B = cos(A + B) + cos(A— B). eo(t) = 2€3(0) = (J3 — 253) cos 2ugt + 2Jod cos(2ive + Wrm)t + J? cos(2uie + Lum )t ~2 Jou €0S(2ure — tm )t + J} COS( Zire — 2itmn)t +(J2 +202) ~ 20? cos 2uymt (284) The last line does not pass through the LP-fiter. Now we compare these coefficients with the corresponding coefficients of an FM signal. We have J} — 25} = (0.99)? — 2(0.10)? = 0.960 (285) 2JoJi = 2(0.99)(0.10) = 0.198 (286) J? = (0.10)? = 0.01 (287) From tabulated values, the closest match is § = 0.4, which is the predicted result. ‘Problem 6. ‘A cartier waveform is frequency-modulated by the sum of two sinusoids: (C) = 100 cos(wet + sinwyyt + 2sin 2w yt), (288) where f. = 100 KHz and fy, = 1 KHz 1) What is the peak frequency deviation from the carrier? b) Estimate the net bandwidth required for transmission of this FM signal. ) Sketch to seale the resulting magnitude line spectrum (one-sided above carrier) [Hint: Express ¢(t) in complex-valued notation and use Bessel functions for series representa- tions to recognize the required coefficients.] [elution] a) af = Max(2%- 4.) (289) = Max (fm CoSwmt + 4 fm COS 2uynt) (290) ‘The maximum value occurs at wx , so that Af =5fm = 5 KHz (291) 78 CL nee | = a {4 tntaryeimet So staner| (293) : a{s | Fda} (204) (298) where By = 1, Ba = 2, w1 = ym, We = 2 = 2m ‘To determine the bandwidth, using a 1% criterion, we find values for which |Jm(8:)Jn(2)| > 0.01 and then determine the resulting bandwidth. Consider a matrix where m is the index of the row and n is the index of the column, We put Jin(1) x Jn(2) in location (m,n) of the matrix. The frequency corresponding to the (m,n) element is equal to 2n +m KHz. (.77)(.58) = 4466 (.77)(.35) = 2695 (.77)(13) = 1001 (.77)(.08) = .0231 (44)(.58) = 2552 (.44)(.35) (.44)(.18) = 0572. +(.44)(.03) = .0132 (41)(.58) = 0688 (.11)(.35) (41)(413) = 0143 (.11)(.03) = .0033 (.02)(.58) = 0116 (.02)(.35) (.02)(.13) = 0026 (,02)(.03) = .0006, ‘The bandwidth is determined by the element with the + sign because it has the maximum frequency. It corresponds to m = 1 and n = 4, resulting in a total bandwidth of: 2[4(2) + 1] = 18 KHz. Problem 6. ‘The analytical method used to find the spectral density of FM with sinusoidal modulation can be used for more general periodic modulating signals with zero mean. Consider the case in which modulating signal is a periodic square wave of unit amplitude and period T. (For convenience, assume that the square wave has even symmetry about the origin). Let the peak frequency deviation from carrier be Aw, and define a modulation index 6 = Aw/wy where wy = 2/7. a) Sketch the instantaneous frequency and phase. b) Derive an expression for the spectral density. c) Sketch the magnitude spectrum for 8 = 79 (298) a(t) = f wi(t)dt = wet + (0) (297) where w; is the instantaneous frequency and Aut r/actc D/A 0) { Aul(T/2)-] T/4 p =3/2 (310) P, = 100 W (unchanged) (311) Py = PJR(8/2) = 100(0.51)? = 26 W (312) B = 2(2)+2(3)=10KHa (313) A certain sinusoid at a frequency of fin Hz is used as a modulating signal in ion of the FM system is set to three times the bandwidth of the AM system. ‘The magnitudes of of those sidebands spaced at -tf;,Hz from carrier in both systems are equal, and the total average powers are equal in both systems. Determine a.) The modulation index of the FM system, and b.) the modulation index of the AM system. 81 Af Toe b,) Let Ay and Ay be the AM and FM carrier peak amplitudes, respectively. We have total average power for FM = total average power for AM. Then, Af =3x (2fm) > 8 = 6. Aj = Al(1 + m?/2) (314) and also magnitudes of sidebands at f. + f,, are equal in both systems, i.e: ‘As|Ja(6)| = Armn/2 (315) From equations (314) and (315) above it. can be seen that the modulation index m is given by: 2\Jx(6)| yi ~ 243(6) [Problem 6.4.3] The output of a given FM modulator with a sinusoidal input is: 8(t) = 0.61 Acos (wt +Bsinwy,t). This output ®(t) is applied to a synchronous detector and an RC lowpass filter, with RC“? = wy. Develop an expression for the average power at the filter output (for an arbitrary A) if it is given that the output is one watt when = 0. (1) can be written as : 8) =A SS Inlf) cos (ue + num). After the synchronous demodulator, assuming that the frequency components around w, are filtered, the output is equal to: a= SS JnlB) cos amt Using equation (6.43) of your book (Stremler) the power across a 1-chm resistor is given by: _ P=8H=F Do nO) ‘The magnitude squared of the RC low-pass filter transfer function is given by: 1 w?, WOE Ty TerRoy tok ‘The average output power of a linear system is given by equation (2.78) of your text (Stremler) as: Pe y |H(nw.) PIP. 82 In this case w, = Wm. Using this relationship we get a Ps at Fx (B)|H (rtm)? = (316) ‘The case of 6 = 0 corresponds to having only the carrier. For 8 = 0, the total power is 1W. Or: a P, = SJ3(0) = 4/8 = 1W (317) replacing in equation (316), 42/8 by 1W we get : p= > BO gory 2B T4n? carrier waveform is phase modulated by a sinusoidal signal f(t). The peak phase deviation is 1 rad when the peak input amplitude is 0.5V. Find the bandwidth using Carson’s rule, and the ratio of the average power in the carrier and the first-order sidebands for each of the following inputs. F(t) = cos 1000m¢ b,) F(t) = 1.2cos300nt ) #(t) = 1.9.c0s200mt For Phase modulation (PM) the modulation index 8 is given by: p= = aky In this case ky = A/a = 1 rad/(0.5 volts) = 2 rad/V The bandwidth B is obtained using Carson’s rule as: B= 2%fm(1+ 8) Let parameter A denote the desired ratio. We have, 5 (8) + 247(8) ROD i a.) fm = 500Hz; 6 = (1)(2) = B=2x (500)(1+2)Hs = 3KHz ‘A = 2.59. From tables of the Bessel functions (1.2)(2) = 2 x (150)(1+ 24)Hz = 1.02K Hz; 18. 83 ©) fn = 100H2; 8 = (1.9)(2) = 8.8; B=2x (100)(1+3.8)Hz = 0.96KHs; A=0.19. [Problem 6.5.4] The testing procedure developed for a certain PM system designed to trans- mit remote sensing information to an orbiting satellite uses the staircase type periodic modulation signal 7(¢) shown in Fig. P-6.5.4 a.) Derive an expression for the spectral density, referred to the carrier frequency we, for this modulation as a function of the peak phase deviation a. (‘The comments in problem 6.3.9 may be helpful here also.) b,) Sketch the magnitude spectrum for a = 1/3 (60°). Solution a.) We can write the PM signal as: Bprg(t) = BAe Med} ‘The function 7(t) is periodic with period T and can be represented by the Fourier series with coefficients F, given by: 1 pts 1 1 (TP sag-inentay yd Rg f dt cm 3 We can also write ®pye(t) as : 7 sog-imuotay 4s L [7 gninginwet, cerimotdt + a fee mente Irs lary Sa(nm/3)[1 + 2cos (a+ 2n7/3)] Sputt) =A So Fy cos (ue + na)t where wo = 2n/T. From this expression the spectral density can be obtained as: Epa (é ASS Faldle-+ ive + mip) + 5 ~ we ~ np) 84 b.) Easy. Problem 6.8.3, Suppose that output of the FM discriminator in Fig 6.25 were applied to a lowpass filter with frequency transfer function H(w), instead of the ideal LPF assumed in equation (6.110). Assuming sinusoidal modulation at w = Wy, show that Eq. (6.115) can be used except that N, must be replaced by Ney? a [ow Pae Sear = Them) Solution:] For a filter with a transfer function H(w), equations (6.109) and (6.110) of the text can be respectively written as : Snel Ne = ge [n/t (uae ‘The output signal s(t) [as in equation (6.100)] for sinusoidal modulation is given by: So(t) = |H(wm)|aky cos (wnt + 8) where @ = /H(wy,). Noting that Aw = aky, the output signal power is then : So = |H (wma) |?(Aw)?/2 ‘Then the output S/1V ratio is then given by: 8. [#(wn)[2(Au)?/2 (/A ge [ou Pao where: 85 Problem 6.8.4) A communication system operates in the presence of white noise with a two sided power spectral density $,(w) = 0.25 x 10-4 W/Hz and with total path losses (including antennas) of 100dB. The input bandwidth is 10-KHz. Calculate the minimum required carrier power of the transmitter for a 10-KHz sinusoidal input of and a 40dB output S/N ratio if the modulation is: a.) AM (DSB-LC), with m=0.707 and m=1.0 b.) PM, with Af = 10KHz and Af = 30KHz c.) PM, with A@ = 1 rad and A@ = Srad. 4.) Summarize your findings by marking these points on a graph of power required (in KW) versus bandwidth (in KHz). Solution:] a.) 2(0.25 x 10-4) W/Ha(2 x 108)Hz = 10-°W Using equation (5.90) from your book (Stremler), Se= [(#) mn] No = 10°8/m? The required transmitter power is then obtained (after taking into account the 100dB losses as): 20KW m= 0.707 b) For Ay = 10KHz, 6 = Af/ fm = 1; Ne = 10~"°W from part (a) Using equation (6.115) from your book (note that $,/N, = 104), we obtain, NS 10-1" at = F5 (104) = 3.33 x 1077 aan, ~ ace?) * From which we obtain the transmitter power as P; = 10!°S, = 3.33KW. For Ay = 30KHz: 8 = 3 and P, =370W c.) Using the results of Example 6.8.1, So) _ (aay? (2) (i) ne" CF) a NeSo (A8PNo for A@ = 3, P; = 10x 10°/(3)? = L.11KW, Se P, = (10"°) =10KW, (Ad=1) d.) Easy. 86 A frequency -division multiplexing system uses SSB-SC subcarrier modula- tion and FM main carrier modulation. There are 20 equal-amplitude voice-input channels, each bandlimited to $.3KHz. A 0.7-KHz guard band is allowed between channels and below the first channel a.) Determine the final transmission bandwidth if the peak frequency deviation is 400K Hz b.) Compute the degradation in signal-to-noise of input No.20 when compared with input No.1 (Assume a white input noise spectral density to the discriminator and no De-emphasis.) c.) Repeat part (b) if PM were used, Solution:] a.) Each channel requires (0.7 +3.3) = 4.0 KHz Snas = (20)(4.0) = 80KHz; Ay = 400KHz; Bog = 5. ‘The final bandwidth is obtained using Carson's rule as: B = 2(80KHz)(1+ 5) = 960KHz. b) Channel 1: 0.7 -4.0KHz Channel 20: 76.7-80KHz. ‘The FM discriminator noise output varies parabolically with frequency: Pao [ 8 a | if 4 y (80)° — (76.7) =~ =i wag = 954.8 = 29.8dB Pi hes? | Lost | (@— (07) c.) For PM, white input spectral noise density gives white output noise. (see example 6.8.1 of your book). Therefore Pio/P; =1 and all channels are treated equally in terms of the noise. Problem 6.10.1] A 10-KHz sinusoidal signal is to be transmitted using FM in the presence of additive white Gaussian Noise. If the S/N improvement at the demodulator output is required to to be 20dB, determine the required peak frequency deviation if a.) no pre-emphasis /de-emphasis is used; and b.) the standard pre-emphasis/de-emphasis is used (cf Fig.6.88) c.) Make a plot of the required peak frequency deviation versus fy, for the conditions in part (b) over the frequency range KHz < fn < 15KHz. Solution] a.) The input and output signal to noise ratio of an an FM discriminator are related by the equation: So _ age Se nN, in this case this relation translates into: 36? = 100, or B= 5.77; Af = Bfm = (6.77)(104) = 57.7KHe b.) With standard preemphasis/deemphasis the signal to noise improvement is governed by the relationship: So gg2pSe; or p'r = 33.898 N. a Where I’ is the S/N improvement due to preemphasis/deemphasis. Assuming f, = 2.1, in equa- tion (6.124) of the book (Stremler), we obtain, T(fm = 10, f; = 2.1) = 10.59, so that 6? = 33.333/10.59 = 3.147 and finally Af = 8 fm = 17.74KH2 c.) Combining the steps in b.) into a more general solution, we get: 10/3 T(fmm) where I’ is given in equation (6.124) of the text. A graph of this result can be easily drawn. Note that in this case the Af is controlled by the highest modulating frequency (assuming equal weighting across the input bandwidth), in contrast to the conditions of (a) in which Af is constant. AS = tm 88

Vous aimerez peut-être aussi